30.07.2015 Views

Logic Games Answers - Ivy Global

Logic Games Answers - Ivy Global

Logic Games Answers - Ivy Global

SHOW MORE
SHOW LESS

Create successful ePaper yourself

Turn your PDF publications into a flip-book with our unique Google optimized e-Paper software.

Preptests 62 <strong>Answers</strong> and Explanations (By <strong>Ivy</strong> <strong>Global</strong>)e) This is the correct answer.5. Type of Question: Local—Must Be TrueWe already know that W can’t be 6 (L must come after it, as per rule 1), P can’t be 6 (G and Smust come after it, as per rule 2), and T can’t be 6 (as per rule 4). Now, we’re told that both Gand S can’t fall on 6, so that leaves only L.b) This is the correct answer.6. Type of Question: <strong>Global</strong>—SubstitutionFor this question we’ll have to find an answer choice that stops T from going in 6 and doesn’tcreate any other restrictions.a) This is the correct answer. We know G and S have to be in 4, 5, or 6, so T will have tocome before at least one of them.b) We’ve seen many cases where T is not immediately before G or S—when T falls in 1.c) We saw L come before T in question 3 c).d) We saw G come before T in question 3 c).e) L could also fall on 6.Questions 7 – 13This is a particularly long and difficult game, but it is manageable if you make a key inference.Rule #3 and #4 combine into an important inference. Y is never with G or O, and without O, we musthave P. This means Y will always occur with P.With this inference, we know we have two windows with R, one window with GP, and onewindow with PY.7. Type of Question: <strong>Global</strong>—ListThis question should be approached with a rule sweep.a) Eliminated by rule # 4: Each window must have O or P.b) This is the correct answer.c) Eliminated by rule #1: Exactly one window must contain G and P.d) Eliminated by rule #3: Y cannot be paired with O.e) Eliminated by rule #2: Exactly two windows have R.8. Type of Question: <strong>Global</strong>—Cannot Be TrueWe should go through this with reference to the rules, past work, and hypotheticals if necessary,so it should be left until the end.a) We saw this in question 13.b) We saw this in question 10.c) This is the correct answer. Each window must have one of either P or O in it.d) We saw this in question 11.e) We saw this in question 9.9. Type of Question: Local—Could Be True<strong>Ivy</strong> <strong>Global</strong>

Hooray! Your file is uploaded and ready to be published.

Saved successfully!

Ooh no, something went wrong!